Download Cardiovascular Blueprint

Survey
yes no Was this document useful for you?
   Thank you for your participation!

* Your assessment is very important for improving the workof artificial intelligence, which forms the content of this project

Document related concepts

Medical ethics wikipedia , lookup

Patient safety wikipedia , lookup

Dysprosody wikipedia , lookup

Electronic prescribing wikipedia , lookup

Adherence (medicine) wikipedia , lookup

List of medical mnemonics wikipedia , lookup

Transcript
Cardiovascular Blueprint
Questions, Answers and Explanations
Question 1
1. Your patient is a 45 year old male with a newly
diagnosed right lower extremity DVT. Which of the
following is the goal INR for him?
A.
B.
C.
D.
1.5-2.5
2.0-3.0
3.0-4.0
3.5-4.5
Answer 1
1. B is the correct answer. Patients with their initial
diagnosis of DVT need to be anti coagulated and have a
goal INR of between 2.0-3.0 if coumadin is used. They
should be anticoagulated for 3-6 months. Consider
repeating the vascular study before discontinuing therapy.
The second DVT should be anticoagulated for life and/or
get a Greenfield filter.
Question 2
2. Your patient is a 54 year old male who presents with bilateral leg pain that
has been present for several months. He is a smoker so your consider
peripheral artery disease as a potential etiology of his symptoms. His ABI
comes back at 1.5. Which of the following is true about this patient's findings?
A. This is a normal ABI
B. This ABI is abnormal.
C. This ABI is inaccurate. Smokers characteristically will have higher ABI's.
This patient needs more sensitive studies such as angiography
D.We treat patients and not diagnostic tests. This patient should be started on
Cilostazol empirically
Answer 2
2. Choice A is the correct answer. An ABI greater than 1.0 is
considered normal and an ABI less than 1.0 is considered
abnormal and suggestive of peripheral artery disease. ABI is
fairly accurate. No clinical data suggests that smokers need
angiography. It is a highly invasive procedure with complications
and should be reserved for those high risk individuals are those
with acute arterial occlusion or those where surgery may be
necessary. Starting patients empirically on Cilostazol is not
appropriate. These medications can cause bleeding and have
side effects.
Question 3
3. Your patient is a 43 year old male that presents with chest pain,
dizziness and shortness of breath. His past medical history is
remarkable for CAD with 2 stents placed 6 months ago. His vital signs
are 52/32, HR-154, SpO2 is 98%, RR-24 Temp-97.3. Which of the
following is likely the cause of this patient's symptoms?
A.
B.
C.
D.
Volume Depletion
Cardiogenic Shock
Septic Shock
CVA
Answer 3
3. Choice B is the correct answer. Cardiogenic shock is defined as
widespread failure of inadequate tissue perfusion resulting in metabolic
demands not being met. Myocardial ischemia is the primary cause.
These patients will likely need to be placed on vasopressors. The main
treatment is to treat the underlying myocardial ischemia first. Volume
depletion can cause tachycardia and hypotension but typically will not
cause you to have chest pain. Patients in septic shock will usually be
febrile. A CVA typically will not be hypotensive or tachycardic.
Question 4
4. Your patient is a 64 year old male that presents with the EKG listed below.
Which of the following is best therapeutic intervention?
A.
B.
C.
D.
Defibrillation
Labetalol
Adenosine
Cardizem
Answer 4
4. Answer D is the correct answer. This patient has
A-Fib with RVR. Defibrillation is reserved for
Ventricular Tachycardia, Ventricular Fibrillation, or
unstable A-Fib. This is a narrow complex tachycardia
that is irregular. Adenosine or Labetalol would be
appropriate for supraventricular tachycardia or sinus
tachycardia.
Question 5
5. Your patient is a 59 year old female newly diagnosed with
primary hypertension. Her blood pressure is 158/96. She has
no other medical problems. Which of the following is the best
choice for first line management for her?
A.
B.
C.
D.
Clonidine
Norvasc
Cardizem
Hydrochlorothiazide
Answer 5
5. Choice D is the correct answer. According to
the JNC guidelines diuretics a and beta blockers
are considered first line for stage I hypertension.
Medications such as clonidine that are centrally
acting alpha blockers have some rebound effect if
stopped abruptly. Calcium channel blockers such
as norvasc and cardizem are considered second
line.
Question 6
6. Your patient is a 84 year old female that presents with a new
onset of systolic murmur best heard at the sternal border. She
has been having near syncope getting progressively worse. She
has no carotid bruits. Which of the following is the most likely
diagnosis?
A.
B.
C.
D.
Aortic Stenosis
Aortic Regurgitation
Carotid Artery Stenosis
Mitral Stenosis
Answer 6
6. Choice A is the correct answer. Aortic Stenosis
presents with systolic murmur. Presenting
symptoms are usually dizziness and sometimes
chest pain. They also can present with CHF.
Mitral Stenosis and Aortic Regurgitation are both
diastolic murmurs so are incorrect. Carotid artery
stenosis is not likely because the patient does not
have any carotid bruits.
Question 7
7. Your patient is a 75 year old male that presents with sharp chest pain
radiating through to his back. His vitals signs are BP 85/53, HR 110, RR 12,
Temp 96.5. His Chem 7 is as follows Na 130, K 4.1, Cl-112, HCO3 22, BUN36, Creatinine-2.5, Glucose 96. Which of the following is the best management
option?
A.
B.
C.
D.
Order a Stat CTA of Chest with IV contrast to look for a dissection
Order a Stat Bedside ECHO
Order a Transesophageal ECHO (TEE)
Call the cardiothoracic surgeon and take the patient to the operating room
immediately.
Answer 7
7. Choice C is the correct answer. This patient with his
presenting symptoms and impaired renal function needs a
TEE. CT with IV contrast is contraindicated when the
creatinine is greater than 2.0. The above two are the only
two options for screening for a dissection. A normal ECHO
is not going to be able give you any information rather the
patient is dissecting or not. Taking the patient to surgery
without a definitive diagnosis is dangerous and should not
be done.
Question 8
8. Which of the following is the medication of choice for
treating Prinzmetal’s Angina?
A. Metoprolol
B. Lisinopril
C. Verapamil
D. Nitroglycerin
Answer 8
8. Choice C is the correct answer. Prinzmetal’s angina or
Variant angina is caused by spasm of the coronary arteries.
It tends to occur at rest. Verapamil best targets it because
it helps dilate arteries. Nitroglycerin has little activity on
arteries. Metoprolol is useful in regular angina because it
helps decrease myocardial demand by blocking the effect
of catecholamines. Lisinopril helps with after load
reduction.
Question 9
9. Which of the following is not recognized as a modifiable
risk factor in the prevention of coronary disease?
A. Diabetes
B. Alcohol intake
C. Smoking
D. Hypertension
Answer 9
9. Choice B is the correct answer. Alcohol
ingestion is not identified as a modifiable risk
factor. In fact mild alcohol intake may be
beneficial in helping heart disease. Diabetes,
smoking, hypertension are modifiable risk
factors.
Question 10
10. Your patient is a 53 year old male that presents with a
dry cough and has been recently started on lisinopril for
hypertension. Which of the following is the best
explanation for the patient's symptoms?
A. Elevated Prostaglandin levels
B. Elevated Bradykinin levels
C. Elevated cyclooxygenase levels
D. Elevated catecholamines
Answer 10
10. Choice B is the correct answer. Elevated bradykinin levels
are responsible for the dry cough in patients that take ACE
inhibitors. It should be thought of as a class effect. Switching the
patient to an Angiotensin Receptor Blocker would help this.
There is no elevated bradykinin levels with that class.
Prostaglandin levels are increased with ACE inhibitors and
ARB’s but do not cause the dry cough. Elevated
cyclooxygenase levels and catecholamine levels have nothing to
do with the dry cough or ACE inhibitors.